Question

1. Find another representation, (r, &theta), for the point under the given conditions. (154),r>0 and 2n<0<47Select the best answer for the question 2. Find all the complex roots of 144(cos 210° + i sin 210°) in polar form. O A. 12(co10. Find a necessary, round your answer to the nearest hundredin O A 622 B.9.60 0.728 D. 16.6820. Sketch the following vector as a position vector and find its magnitude: v = 91 + 12j V = 21 1 y + + - 12 - - 4 A 4 8 12

0 0
Add a comment Improve this question Transcribed image text
Answer #1

Answer to the above questions -

1. Given point (1, π/3)

since θ is in range 2π < θ < 4π

option C. (1, 7π/3)

2.

option C.

12 (cos 105 + i sin105) , 12 (cos 285 + i sin 285)

3. option D. 28 cans

4.

option C. 40 ft.lb

5.

option D. a = 16.68

ДАВС ath & tan 45o = 1 aь с 61 АвсТ 6/ ten 36° сь Б: 61 xo. 226 - 44. 32 а: 61 — ь - 61 - 44.32 - 16.68

According to Chegg's policy, I am obliged to answer only the first four questions per post. The rest of them you can post as a separate question.

Please comment if you have any queries related to the answer.
Please UPVOTE if the answer was helpful.

Add a comment
Know the answer?
Add Answer to:
1. Find another representation, (r, &theta), for the point under the given conditions. (154),r>0 and 2n<0<47...
Your Answer:

Post as a guest

Your Name:

What's your source?

Earn Coins

Coins can be redeemed for fabulous gifts.

Not the answer you're looking for? Ask your own homework help question. Our experts will answer your question WITHIN MINUTES for Free.
Similar Homework Help Questions
ADVERTISEMENT
Free Homework Help App
Download From Google Play
Scan Your Homework
to Get Instant Free Answers
Need Online Homework Help?
Ask a Question
Get Answers For Free
Most questions answered within 3 hours.
ADVERTISEMENT
ADVERTISEMENT
ADVERTISEMENT